Proof by Induction with Exponents

Click For Summary
The discussion focuses on proving by mathematical induction that 4 divides the expression 7^n - 3^n for n ≥ 1. The base case is established correctly with P(1): 7 - 3 = 4. Participants suggest rewriting the expression in terms of modular arithmetic to simplify the proof. There is confusion about notation, clarifying that "4 | (7^n - 3^n)" indicates divisibility, not division. The conversation emphasizes the need for careful manipulation and factoring to complete the induction step.
Texans80mvp
Messages
1
Reaction score
0

Homework Statement



By mathematical induction, prove that for n ≥ 1, 4/(7n - 3n).

Homework Equations


The Attempt at a Solution



I got the base case down P(1): 7-3=4.

Now the actual problem,

7n - 3n = 4x
7n+1 - 3n+1 = 7(7n) - 3(3n)
=7(4x + 3n) - 3(7n - 4x)
=21x+ (7(3n)) - (3(7n)) + 12x

-This is the point at which I get stuck there is nothing I can really factor out and I'm pretty sure I messed up earlier or there is something I have to do with the 7 and 3n. Any help would be appreciated.

Trying to get to: 7n+1 - 3n+1
 
Physics news on Phys.org
Here is how I would go about it. Since we care about 7^n and 3^n mod 4, it might be helpful to write 7^n=4a+r and 3^n=4b+r where r is the remainder when 7^n (or 3^n) is divided by 4 (Why must r be the same for both expressions?)
Write 7^(n+1)=7*7^n and 3^(n+1)=3*3^n and substitute.
 
Last edited:
Texans80mvp said:

Homework Statement



By mathematical induction, prove that for n ≥ 1, 4/(7n - 3n).



Homework Equations





The Attempt at a Solution



I got the base case down P(1): 7-3=4.

Now the actual problem,

7n - 3n = 4x
7n+1 - 3n+1 = 7(7n) - 3(3n)
=7(4x + 3n) - 3(7n - 4x)

Try not substituting that second term so you have$$
7(4x+3^n) - 3\cdot 3^n$$and see if you can factor out a 4 from that.
 
Texans80mvp said:

Homework Statement



By mathematical induction, prove that for n ≥ 1, 4/(7n - 3n).

You probably meant to write 4|(7^n-3^n). Not '/'. The '|' means 'divides'.
 
Question: A clock's minute hand has length 4 and its hour hand has length 3. What is the distance between the tips at the moment when it is increasing most rapidly?(Putnam Exam Question) Answer: Making assumption that both the hands moves at constant angular velocities, the answer is ## \sqrt{7} .## But don't you think this assumption is somewhat doubtful and wrong?

Similar threads

  • · Replies 3 ·
Replies
3
Views
2K
  • · Replies 48 ·
2
Replies
48
Views
5K
  • · Replies 15 ·
Replies
15
Views
3K
  • · Replies 2 ·
Replies
2
Views
2K
  • · Replies 4 ·
Replies
4
Views
8K
  • · Replies 11 ·
Replies
11
Views
2K
  • · Replies 8 ·
Replies
8
Views
3K
  • · Replies 2 ·
Replies
2
Views
2K
  • · Replies 3 ·
Replies
3
Views
9K
Replies
18
Views
2K